Why A and Not B?
I thought B would make more sense
Eugene on January 20, 2023
  • June 1999 LSAT
  • SEC1
  • Q11
1
Reply
Why is D wrong?
I approached this question as a main point question, however, LSAT is claiming it is a MBT questi...
Abigail-Okereke on October 19, 2022
  • June 1999 LSAT
  • SEC1
  • Q18
1
Reply
B
I can't understand why B is wrong. Does anyone have any input?
AndrewArabie on September 30, 2022
  • June 1999 LSAT
  • SEC1
  • Q21
2
Replies
Why is B incorrect?
Please explain
odsimkins on August 25, 2020
  • June 1999 LSAT
  • SEC1
  • Q17
1
Reply
Could someone please explain this answer? I pi...
Could someone please explain this answer? I picked E and It was wrong. Thanks
jingjingxiao11111@gmail.com on May 19, 2020
  • June 1999 LSAT
  • SEC1
  • Q22
1
Reply
Help
This is a strengthen with sufficient question, isn't it? To figure out this type of question, you...
Batman on May 19, 2020
  • June 1999 LSAT
  • SEC1
  • Q24
4
Replies
can someone please explain the right answer?
can someone please explain the right answer? thanks
jingjingxiao11111@gmail.com on May 18, 2020
  • June 1999 LSAT
  • SEC1
  • Q17
1
Reply
can someone please explain right answer?
can someone explain right answer please? thanks
jingjingxiao11111@gmail.com on May 17, 2020
  • June 1999 LSAT
  • SEC1
  • Q16
1
Reply
B and C
Can you please tell the difference between B & C?
RitaW on May 12, 2020
  • June 1999 LSAT
  • SEC1
  • Q12
2
Replies
Help
I don't understand why the answer goes to (A), not (B).
Batman on May 12, 2020
  • June 1999 LSAT
  • SEC1
  • Q3
2
Replies
PT 28, S1, Q23
Could you please explain why A is the correct answer? Thank you!
smilde11 on July 27, 2018
  • June 1999 LSAT
  • SEC1
  • Q23
1
Reply
Explanation
Could you explain this? Thanks
Jborj653 on July 16, 2015
  • June 1999 LSAT
  • SEC1
  • Q15
1
Reply
Explanation
Could you explain this? Thanks
Jborj653 on July 15, 2015
  • June 1999 LSAT
  • SEC1
  • Q13
1
Reply
Help
Could you please explain why the answer goes to (d)? And, why (b) can't be the answer? Thanks,
Batman on December 6, 2014
  • June 1999 LSAT
  • SEC1
  • Q25
1
Reply